Lista III

Cálculo C

Exercício 1   Calcule os seguintes limites:
1.
$\lim\limits_{n\to \infty} \frac{n}{n+1}$
2.
$ \lim\limits_{n\to \infty} \frac{n+3}{n^3+4}$.
3.
$\lim\limits_{n\to \infty} \frac{n!}{n^n}.$

Exercício 2  

Encontre os planos tangentes às superfícies nos pontos indicados:

1.
S definida pela equação $x^2+2y^2+3xz=10,\quad P_0=(1,2,1/3)$
2.
S gráfico de $f(x,y)=(\cos x)(\cos y),\quad P_0=(0,\pi/2)$

Exercício 3   Suponha que uma montanha tenha a forma de um parabolóide elítico de equação

z=c-ax2-by2,

onde a,b e c são constantes positivas e x e y são as direções leste-oeste e norte-sul. No ponto (1,1) qual a direção para a qual a altura aumenta mais rapidamente? Se uma pedra fosse solta em (1,1,c-a-b) qual a direção que começaria a rolar?

Exercício 4   Suponha que uma partícula seja ejetada de uma superfície de equação x2+y2-z2=1 do ponto $(1,1,\sqrt{3})$ na direção da normal que aponta para o plano XY, com velocidade 10 m/s. Determine quando e onde a partícula encontrará o plano XY.

Exercício 5   Um barco se move na direção norte-leste com velocidade escalar 20 km/h. Assuma que a temperatura decresce a uma taxa de 0.2oC/km na direção norte e 0.3oC/km na direção leste. Qual é a taxa de variação da temperatura observada pelo barco?

Exercício 6 (Método do steepest descent)   Suponhamos que estejamos interessados em encontrar a solução de

f(x,y,z)=0.

Seja então P0 uma primeira aproximação. Então se f(P0)=c0> 0podemos nos aproximar da solução escolhendo uma direção na qual f decresça, de fato aquela que f decresça mais rapidamente seria a ideal. Isto nos sugere escolher

\begin{displaymath}\mathbf{v}=-\boldsymbol{\nabla} f(P_0)\end{displaymath}

e tomando h pequeno podemos tomar como segunda aproximaç ão

\begin{displaymath}P_1=P_0-h\boldsymbol{\nabla} f(P_0).\end{displaymath}

o processo continua até termos $f(P_n)\leq \epsilon$.

Aplique este método conhecido como método do gradiente descendente para encontrar a solução de


\begin{align*}2x+3y+4z&=5\\
x^2+y^2+z^2&=7\\
xyz&=4
\end{align*}
Sugestão:Aplique o método para a função:

f(x,y,z)=(2x+3y+4z-5)2+(x2+y2+z2-7)2+(xyz-4)2.

Exercício 7   Seja $\rho=\rho(x,y,z,t)$ a densidade de um fluido em movimento que se desloca com a seguinte velocidade

V(x,y,z,t)

isto é V denota a velocidade da partícula do fluido que no instante t estava em (x,y,z). Determine

\begin{displaymath}\frac{d\rho}{dt}\end{displaymath}

e explique sua resposta através de um raciocínio físico e geométrico.

Exercício 8   Uma partícula desliza sem frição ao longo da hélice representada pelas equações paramétricas:

\begin{align*}x&=R\cos \theta\\
y&=R{\text {\ sen }}\theta\\
z&=\rho \theta
\end{align*}

Se a partícula começa no ponto que corresponde a z=z0>0, então quando ela atinge a altura z sua velocidade é dada por

\begin{displaymath}v(t)=\frac{ds}{dt}=\sqrt{(z_0-z)2g}.\end{displaymath}

1.
Encontre o comprimento da parte da hélice entre os plano z=z0 e z=z1com 0< z1<z0.
2.
Calcule o tempo T0 que leva para a partícula alcançar o plano z=0.

Exercício 9   Verifique as seguintes identidades:
1.
$\text{rot}(f\ensuremath{ \boldsymbol{F}} )=f\,\text{rot}\ensuremath{ \boldsymbol{F}} +\ensuremath{ \boldsymbol{\nabla}} f\times \ensuremath{ \boldsymbol{F}} $
2.
$\text{div}(f\ensuremath{ \boldsymbol{F}} )=f\,\text{div}\ensuremath{ \boldsymbol{F}} +\ensuremath{ \boldsymbol{F}}\cdot \ensuremath{ \boldsymbol{\nabla}} f$
3.
$\text{div} \,\text{rot} \ensuremath{ \boldsymbol{F}} =0.$
4.
$\text{rot}\,\ensuremath{ \boldsymbol{\nabla}} f=\ensuremath{ \mathbf{0}} .$
5.
$\text{div}(\ensuremath{ \boldsymbol{F}}\times\ensuremath{ \boldsymbol{G}} )=
\e...
...F}} -\ensuremath{ \boldsymbol{F}}\cdot \text{rot}\ensuremath{ \boldsymbol{G}} .$

Exercício 10   Uma esfera de raio 10 cm com centro em (0,0,0) roda com velocidade angular $\omega=4$ com eixo de rotação $\ensuremath{ \mathbf{z}} $ e sentido anti-horário no plano $\ensuremath{ \mathbf{x}}\ensuremath{ \mathbf{y}} $.
1.
Encontre o vetor de rotação $\ensuremath{ \boldsymbol{\omega}} $.
2.
Encontre a velocidade $\ensuremath{ \mathbf{v}} =\ensuremath{ \boldsymbol{\omega}}\times \ensuremath{ \mathbf{r}} $ quando $\ensuremath{ \mathbf{r}} =5\sqrt{2}(\ensuremath{ \mathbf{i}} -\ensuremath{ \mathbf{j}} )$.
3.
Encontre a velocidade do ponto $(0,5\sqrt{3},5)$ na esfera.

Exercício 11   Seja $\ensuremath{ \boldsymbol{F}}\colon \ensuremath {\ensuremath{ \hbox{\matii R}}^3 }\rightarrow\ensuremath {\ensuremath{ \hbox{\matii R}}^3 } $ um campo conservativo

\begin{displaymath}\ensuremath{ \boldsymbol{F}} (\ensuremath{ \mathbf{x}} )=
-\e...
...l{\nabla}}\ensuremath{ \mathbf{V}} (\ensuremath{ \mathbf{x}} ).\end{displaymath}

Uma curva $\phi\colon \ensuremath {\ensuremath{ \hbox{\matii R}}}\rightarrow \ensuremath {\ensuremath{ \hbox{\matii R}}^3 } $ denomina-se quase-Newtoniana se existem constantes mi, chamadas suas componentes de massa tais que

\begin{displaymath}F_i(\phi(t))=m_i\ddot{\phi_i}(t)\end{displaymath}

para cada i=1,2,3. Isto é, relativamente à direção $\ensuremath{ \mathbf{i}} $, ela se comporta como se tivesse massa mi. Defina sua energia cinética $\ensuremath{ \boldsymbol{K}} (t)$ e sua energia potencial $\ensuremath{ \boldsymbol{P}} (t)$ no tempo t por


\begin{align*}\ensuremath{ \boldsymbol{K}} (t)&=\frac{1}{2}\mathop{\pmb{\sum}}\l...
...remath{ \boldsymbol{P}} (t)&=\ensuremath{ \boldsymbol{V}} (\phi(t))
\end{align*}
Prove que a lei de conservação de energia é válida para partículas quase-Newtonianas, isto é, $\ensuremath{ \mathbf{K}} +\ensuremath{ \mathbf{P}} =\text{const}$ .

Sugestão: Derive $ \ensuremath{ \mathbf{K}} +\ensuremath{ \mathbf{P}} $.

Exercício 12   Encontre as expressões em coordenadas cilíndricas para o divergente e o rotacional

Exercício 13   Definimos o operador de Laplace $\ensuremath{ \boldsymbol{\nabla}} ^2$(ou Laplaciano), que opera sobre funções escalares f, como o divergente do gradiente:

\begin{displaymath}\ensuremath{ \boldsymbol{\nabla}} ^2\,f=\ensuremath{ \boldsym...
...tial^2\,f}{\partial\,y^2}+
\frac{\partial^2\,f}{\partial\,z^2}
\end{displaymath}

Determine a expressão em coordenadas cilíndricas para o Laplaciano.

Exercício 14   Mostre que se um campo é conservativo, então não admite linhas de campo fechadas (ou trajetórias fechadas) isto é, $\gamma(t)$tal que

\begin{displaymath}\gamma'(t)=\ensuremath{ \boldsymbol{F}} (\gamma(t))\quad \forall\,t\in\ensuremath {\ensuremath{ \hbox{\matii R}}^n }\end{displaymath}

com $\gamma(T)=\gamma(0)$ e T>0. Uma tal trajetória denomina-se periódica. Ver figura 1 abaixo.





 
Figure 1: Campo conservativo
\includegraphics[width=0.60\textwidth]{conserv.eps}

Aldrovando Azeredo Araujo